please help i dont know how to do this

Please Help I Dont Know How To Do This

Answers

Answer 1

Answer:

CD: 8.5

m<D: 20.6°

m<C: 69.4°

Step-by-step explanation:

CD:

The first thing it wants us to do is find the length of CD. Since the triangle shown is a right triangle, we can use Pythagorean theorem ([tex]a^{2}+b^{2}=c^{2}[/tex])  to solve for the missing length. It's important to remember that when using the Pythagorean theorem, c is the hypotenuse.

[tex]a^{2}+b^{2}=c^{2}\\3^{2}+8^{2}=c^{2}\\9 + 64 = c^{2}\\73 = c^{2}\\\sqrt{73} =c[/tex]

Since our answer is no an integer, we must turn it into a decimal.

[tex]\sqrt{73}[/tex] ≈ 8.544003745 ≈ 8.5

m<D:

Now, they want us to find the measure of <D. To do this, we will need to use trig functions (sine, cosine, tangent). To help us determine which trig function to use, we can remember the acronym SOH CAH TOA. This acronym tells us that sine is equal to opposite divided by hypotenuse, cosine is equal to adjacent divided by hypotenuse, and tangent is equal to opposite divided by adjacent. Since we do the hypotenuse and sides adjacent and opposite of <D, we can choose whichever trig function we want. For this problem, we will use tangent, so we can avoid using a rounded number, 8.5, as one of our sides.

Tan(D) = opposite / adjacent

Tan(D) = 3 / 8     [Take the tan inverse of both sides}

[tex]Tan^{-1}(Tan(D))=Tan^{-1}(3/8)[/tex]     [Simplify]

[tex]D=Tan^{-1}(3/8)[/tex]     [Solve]

D ≈ 20.55604522

D ≈ 20.6°

m<C:

Lastly, we must find the last unknown angle on the triangle. Since all angles on a triangle total 180°, if know that <C+<D+<E=180°. Let's solve this equation.

<C+<D+<E=180°

<C + 20.6 + 90 = 180     [Add]

<C + 110.6 = 180     [Subtract]

<C = 180 - 110.6     [Solve]

<C = 69.4°

If this answer helped you, please leave a thanks!

Have a GREAT day!!!


Related Questions

Point C has the same y-coordinate as point B and the distance between point B and point C is equal to
the distance between point C and the y-axis. Point A has the same x-coordinate as point C and the
distance between point A and point C is twice the distance between point B and point C.
What is one possible location of point A?
How many possible locations are there for point A?
12
A?

Answers

We can conclude that point A is located at the origin (0, 0).

There is only one possible location for point A is at the origin.

Let's revisit the given information to determine the possible location of point A.

Point C has the same y-coordinate as point B.

This means that the y-coordinate of point C is equal to the y-coordinate of point B.

The distance between point B and point C is equal to the distance between point C and the y-axis.

Let's assume the distance between point B and point C is represented by "d".

According to the information given, the distance between point C and the y-axis is also "d".

Point A has the same x-coordinate as point C.

This implies that the x-coordinate of point A is equal to the x-coordinate of point C.

The distance between point A and point C is twice the distance between point B and point C.

Let's assume the distance between point B and point C is represented by "d".

According to the information given, the distance between point A and point C is 2d.

Based on this information, we can analyze the relationships between the points:

Since the distance between point B and point C is equal to the distance between point C and the y-axis, we can infer that point B lies on the y-axis.

The x-coordinate of point B is 0.

As point C has the same y-coordinate as point B, the y-coordinate of point C is also determined to be the same as the y-coordinate of point B.

Since point A has the same x-coordinate as point C, the x-coordinate of point A will also be 0.

The distance between point A and point C is twice the distance between point B and point C.

As the distance between point B and point C is "d", the distance between point A and point C is 2d.

For similar questions on possible location

https://brainly.com/question/18181905

#SPJ11

ہے
x
-3
-2
0
2
3
1(x)
9
4
0
4
9
What is the domain of this function?
OA. (-3,9)
OB. (-3, -2, 0, 2, 3)
OC. {0, 4, 9)
OD. (0, 2, 3)

Answers

Answer:

introduction of a business invironment

Which statement about rectangles is true?
1. Only some rectangles are parallelograms.
2. Parallelograms have exactly 1 pair of parallel sides.
3. So, only some rectangles have exactly 1 pair of parallel sides.
1. All rectangles are parallelograms.
2. Parallelograms have 2 pairs of parallel sides.
3. So, all rectangles have 2 pairs of parallel sides.
1. Only some rectangles are parallelograms.
2. Parallelograms have 2 pairs of parallel sides.
3. So, only some rectangles have 2 pairs of parallel sides.
1. All rectangles are parallelograms.
2. Parallelograms have exactly 1 pair of parallel sides.
3. So, all rectangles have exactly 1 pair of parallel sides.

Answers

The correct statement about rectangles is:

1. All rectangles are parallelograms.

2. Parallelograms have exactly 1 pair of parallel sides.

3. So, all rectangles have exactly 1 pair of parallel sides.

A rectangle is a type of parallelogram that has additional properties. By definition, a rectangle is a quadrilateral with four right angles. This means that opposite sides of a rectangle are parallel. Since all four sides of a rectangle are right angles, it follows that a rectangle has exactly 1 pair of parallel sides.

Option 1 states that only some rectangles are parallelograms, which is incorrect. All rectangles are parallelograms because they have opposite sides that are parallel.

Option 2 states that parallelograms have 2 pairs of parallel sides, which is also incorrect. Parallelograms have exactly 2 pairs of parallel sides, not 4. A rectangle is a special type of parallelogram that has additional properties such as all angles being right angles.

Option 3 states that only some rectangles have 2 pairs of parallel sides, which is incorrect. All rectangles have exactly 1 pair of parallel sides, not 2. Having 2 pairs of parallel sides would make a shape a parallelogram, not a rectangle.

Therefore, the correct statement is that all rectangles are parallelograms and have exactly 1 pair of parallel sides. 1,2,3 are correct.

For more suhc questions on rectangles visit:

https://brainly.com/question/25292087

#SPJ8

Question 5 of 10
The heights of 200 adults were recorded and divided into two categories.
Male
Female
6' or over
13
4
Tunder
Under 6'
85
Which two-way frequency table correctly shows the marginal frequencies?

Answers

The two-way frequency table that correctly shows the marginal frequencies is Table B.

What is a two-way frequency table?

A two-way frequency table is used to display the frequencies of two categories of variables collected from a single group. We are asked to find the two-way frequency table that correctly shows the marginal frequencies and to do this, we can see that in the second table, the sum of men who participated in the survey amounted to

12 + 86 = 98

Now the total number of adults that participated in the survey was 200. This means that the number of women was 102. That is;

200 - 98 = 102

The only table that represents the number of women as 98 is table B and this makes it the correct choice.

Learn more about a two-way frequency table here:

https://brainly.com/question/16148316

#SPJ1

Washington Junior High is holding a bake sale to raise money for new computers in their library. The principal has estimated that they will need 4 1/2 dozen cookies. If 9 parents have volunteered to bring cookies, how many cookies will each need to bring.​

Answers

Answer: 6 cookies

Step-by-step explanation:

4.5 dozen is 54 cookie

Dividen 54 by the 9 parents is 6 cookie so each partner need to make at least 6 cookie

NO LINKS!! URGENT HELP PLEASE!!

#23 & 24: Please help me ​

Answers

Answer:

[tex]\textsf{23)} \quad y=3\left(\dfrac{5}{3}\right)^x[/tex]

[tex]\textsf{24)} \quady=2\left(\dfrac{1}{2}\right)^x[/tex]

Step-by-step explanation:

The general formula for an exponential function is:

[tex]\boxed{\begin{minipage}{9 cm}\underline{General form of an Exponential Function}\\\\$y=ab^x$\\\\where:\\\phantom{ww}$\bullet$ $a$ is the initial value ($y$-intercept). \\ \phantom{ww}$\bullet$ $b$ is the base (growth/decay factor) in decimal form.\\\end{minipage}}[/tex]

Question 23

From inspection of the given graph, the exponential curve passes through the points (0, 3) and (1, 5).

The y-intercept is the value of y when x = 0. Therefore, a = 3.

To find the value of b, substitute point (1, 5) and the found value of a into the exponential function formula:

[tex]\begin{aligned}y&=ab^x\\\\\implies 5&=3b^1\\\\5&=3b\\\\b&=\dfrac{5}{3}\end{aligned}[/tex]

To write an equation for the graphed exponential function, substitute the found values of a and b into the formula:

[tex]\boxed{y=3\left(\dfrac{5}{3}\right)^x}[/tex]

[tex]\hrulefill[/tex]

Question 24

From inspection of the given graph, the exponential curve passes through points (-1, 4) and (0, 2).

The y-intercept is the value of y when x = 0. Therefore, a = 2.

To find the value of b, substitute point (-1, 4) and the found value of a into the exponential function formula:

[tex]\begin{aligned}y&=ab^x\\\\\implies 4&=2b^{-1}\\\\2&=b^{-1}\\\\2&=\dfrac{1}{b}\\\\b&=\dfrac{1}{2}\end{aligned}[/tex]

To write an equation for the graphed exponential function, substitute the found values of a and b into the formula:

[tex]\boxed{y=2\left(\dfrac{1}{2}\right)^x}[/tex]

NO LINKS!! URGENT HELP PLEASE!!

a. Discuss the association.

b. Predict the amount of disposable income for the year 2000.

c. The actual disposable income for 2000 was $8,128 billion. What does this tell you about your model?​

Answers

Answer:

a) See below.

b) $911 billion

c) See below.

Step-by-step explanation:

 

Linear regression is a statistical technique used to model the relationship between a dependent variable and one or more independent variables by fitting a linear equation to the observed data.

It estimates the slope and y-intercept of a straight line that minimizes the overall distance between the observed data points and the predicted values. The linear regression equation is y = ax + b.

Part a

The association between year and amount of disposable income is indicated by the linear regression equation y = ax + b.

The value of a is the slope of the linear regression line, and represents the average rate of change in disposable income per year. As a = 14.0545, it means that the disposable income increases by approximately $14.0545 billion dollars each year.  

As the value of r (correlation coefficient) is very close to +1, it indicates a very strong positive linear correlation between the year and disposable income. This suggests that as the years progress, the disposable income tends to increase.

Part b

Linear regression equation:

[tex]\boxed{y=14.05454545x-27198}[/tex]

To predict the amount of disposable income for the year 2000, we can substitute x = 2000 into the linear regression equation:

[tex]y = 14.05454545 \cdot 2000 - 27198[/tex]

[tex]y=28109.0909...-27198[/tex]

[tex]y=911.0909...[/tex]

[tex]y=911[/tex]

Therefore, the predicted amount of disposable income for the year 2000 is approximately $911 billion.

Part c

The predicted value of $911 billion for the year 2000 is significantly lower than the actual value of $8128 billion. This implies that the model is not accurately capturing the increasing trend in disposable income over time, leading to an underestimation of the income level in 2000. This suggests that the model may have limitations or inaccuracies when extrapolating beyond the range of the provided data. It indicates the need for caution and further analysis when using the model to make predictions outside of the given timeframe.

A company is trying to determine the crossover point for two different manufacturing processes. Process A has a fixed cost of $50,000 and a variable cost of $10 per unit produced. Process B has a fixed cost of $30,000 and a variable cost of $15 per unit produced. The selling price for the product is $25 per unit. The company wants to know at what production volume the total cost of each process will be equal, and which process should be chosen above that volume.

Answers

Answer:

To determine the crossover point, we need to find the production volume at which the total cost of each process is equal. Let's denote the production volume as "x."

For Process A:

Fixed cost = $50,000

Variable cost = $10 per unit

Total cost for Process A = Fixed cost + (Variable cost * x)

For Process B:

Fixed cost = $30,000

Variable cost = $15 per unit

Total cost for Process B = Fixed cost + (Variable cost * x)

The selling price for the product is $25 per unit.

To find the crossover point, we'll equate the total costs of both processes and solve for "x."

Total cost for Process A = Total cost for Process B

$50,000 + ($10 * x) = $30,000 + ($15 * x)

Now, let's solve this equation to find the crossover point:

$50,000 + $10x = $30,000 + $15x

$10x - $15x = $30,000 - $50,000

-$5x = -$20,000

x = -$20,000 / -$5

x = 4,000

Therefore, the crossover point occurs at a production volume of 4,000 units.

To determine which process should be chosen above that volume, we'll compare the total costs of each process at a production volume greater than 4,000 units.

For Process A:

Total cost for Process A = $50,000 + ($10 * x)

Total cost for Process A = $50,000 + ($10 * 4,000)

Total cost for Process A = $50,000 + $40,000

Total cost for Process A = $90,000

For Process B:

Total cost for Process B = $30,000 + ($15 * x)

Total cost for Process B = $30,000 + ($15 * 4,000)

Total cost for Process B = $30,000 + $60,000

Total cost for Process B = $90,000

At a production volume greater than 4,000 units, both Process A and Process B have the same total cost of $90,000. Therefore, either process can be chosen above that volume without any cost advantage.

foreign direct investment helps improve the economic situation of a recipient country by increasing —- opportunities in the country that the company invests in.

Answers

Foreign direct investment helps improve the economic situation of a recipient country by increasing employment opportunities in the country that the company invests in.

When foreign companies invest in a recipient country, they often establish or expand their operations, which requires hiring local workers. This leads to job creation and reduces unemployment rates in the recipient country.

Increased employment opportunities result in more individuals having access to income and improved standards of living.

Foreign direct investment also contributes to the transfer of technology, knowledge, and skills to the recipient country. Multinational companies often bring advanced technologies, production techniques, and management practices that may not have been available or widely adopted in the recipient country.

Furthermore, foreign direct investment stimulates domestic investment and encourages the growth of local businesses. When foreign companies invest in a recipient country, they often form partnerships or engage in supply chain relationships with local firms.

Overall, foreign direct investment increases employment opportunities, fosters technology transfer, and stimulates domestic investment, all of which contribute to improving the economic situation of a recipient country.

for similar questions on company invests.

https://brainly.com/question/27717275

#SPJ8

find the inverse of the matrix

[1 0 0

1 1 0

1 1 1 ]

please show and explain each step

Answers

The inverse of the given matrix [1 0 0; 1 1 0; 1 1 1] is:

[1 0 0]

[-1 1 0]

[0 -1 1]

To find the inverse of a matrix, we can follow these steps:

Step 1: Write the given matrix and the identity matrix side by side.

[1 0 0 | 1 0 0]

[1 1 0 | 0 1 0]

[1 1 1 | 0 0 1]

Step 2: Apply row operations to transform the given matrix into the identity matrix on the left side.

Subtract the first row from the second row: R2 = R2 - R1

[1 0 0 | 1 0 0]

[0 1 0 | -1 1 0]

[1 1 1 | 0 0 1]

Subtract the first row from the third row: R3 = R3 - R1

[1 0 0 | 1 0 0]

[0 1 0 | -1 1 0]

[0 1 1 | -1 0 1]

Subtract the second row from the third row: R3 = R3 - R2

[1 0 0 | 1 0 0]

[0 1 0 | -1 1 0]

[0 0 1 | 0 -1 1]

Step 3: The matrix on the right side is now the inverse of the given matrix. Therefore, the inverse of the given matrix is:

[1 0 0]

[-1 1 0]

[0 -1 1]

The inverse of the given matrix [1 0 0; 1 1 0; 1 1 1] is:

[1 0 0]

[-1 1 0]

[0 -1 1]

For more questions on matrix

https://brainly.com/question/2456804

#SPJ8

What is the domain of the square root function graphed below?

On a coordinate plane, a curve open up to the right in quadrant 4. It starts at (0, negative 1) and goes through (1, negative 2) and (4, negative 3).
x less-than-or-equal-to negative 1
x greater-than-or-equal-to negative 1
x less-than-or-equal-to 0
x greater-than-or-equal-to 0
Mark this and return

Answers

The domain of the square root function is x greater-than-or-equal-to 0, since the function is defined for all non-negative x-values or x-values greater than or equal to zero.

The domain of the square root function graphed below can be determined by looking at the x-values of the points on the graph.

From the given information, we can see that the curve starts at (0, -1) and goes through (1, -2) and (4, -3).

The x-values of these points are 0, 1, and 4.

Since the square root function is defined for any non-negative x-values or x-values more than or equal to zero, its domain is x greater-than-or-equal-to 0.

for such more question on domain

https://brainly.com/question/16444481

#SPJ8

Roberto made a line plot to show the weight in pounds of the bags of granola in his store he concluded that the total weight of the granola was 2/1/2+2/3/4+3=8/1/4 pounds

Answers

The correct total weight of the bags of granola is 8 1/4 pounds.

One thing that can be done to improve Roberto's reasoning is to ensure the accuracy of the calculations.

In his conclusion, Roberto added the weights of the bags of granola (2 1/2, 2 3/4, and 3) and claimed that the total weight was 8 1/4 pounds. However, the sum of these weights does not equal 8 1/4 pounds.

To address this, Roberto should recheck his calculations. Adding mixed numbers involves adding the whole numbers separately and then adding the fractions separately. In this case, 2 1/2 + 2 3/4 + 3 can be calculated as follows:

2 + 2 + 3 = 7 (sum of whole numbers)

1/2 + 3/4 = 2/4 + 3/4 = 5/4 = 1 1/4 (sum of fractions)

Thus, the correct sum is 7 + 1 1/4 = 8 1/4 pounds.

By double-checking the calculations and providing the accurate sum, Roberto's reasoning would be more precise, reliable, and free from errors.

For more such questions on granola,click on

https://brainly.com/question/27741276

#SPJ8

The probable question may be:

Roberto made a line plot to show the weight in pounds of the bags of granola in his store he concluded that the total weight of the granola was 2 1/2+2 3/4+3=8 1/4 pounds.

what is one thing that you could do to Roberto's Reasoning

I need help , any of u guys have the answer?

Answers

the answer the third choice
(x to the power of 2+1)+(5-x)
first, u keep x to the power of two because there are no like terms, and then you combine the like terms 1 and 5 to get 6, then you subtract x

therefore, the answer is x squared minus x plus 6

3. In ∆ JAM, which of the following statement is always TRUE?

Answers

The option that shows the missing angles in the triangle is:

Option C: m∠1 < m∠4

How to identify the missing angle?

We know that the sum of angles in a triangle is 180 degrees.

Therefore looking at the given triangle, we can say that:

m∠1 + m∠2 + m∠3 = 180°

We also know that the sum of angles on a straight line is 180 degrees and as such we can say that:

m∠3 +  m∠4 = 180°

By substitution we can say that:

m∠4 = m∠1 + m∠2

Thus:

m∠1 < m∠4

Read more about Missing Angle at: https://brainly.com/question/28293784

#SPJ1

The missing options are:

m∠1 > m∠4

m∠2 > m∠4

m∠1 < m∠4

m∠3 = m∠4

which equation represents the slope intercept form of the line when the y intercept is (0,-6) and the slope is -5

Answers

The values into the slope-intercept form, we have y = -5x - 6

The slope-intercept form of a linear equation is given by:

y = mx + b

where 'm' represents the slope of the line, and 'b' represents the y-intercept.

In this case, the y-intercept is (0, -6), which means that the line crosses the y-axis at the point (0, -6).

The slope is given as -5.

Therefore, substituting the values into the slope-intercept form, we have:

y = -5x - 6

This equation represents the line with a y-intercept of (0, -6) and a slope of -5.

for such more question on slope-intercept form

https://brainly.com/question/11990185

#SPJ8

Which statement about rectangles is true?
1. Only some rectangles are parallelograms.
2. Parallelograms have exactly 1 pair of parallel sides.
3. So, only some rectangles have exactly 1 pair of parallel sides.
1. All rectangles are parallelograms.
2. Parallelograms have 2 pairs of parallel sides.
3. So, all rectangles have 2 pairs of parallel sides.
1. Only some rectangles are parallelograms.
2. Parallelograms have 2 pairs of parallel sides.
3. So, only some rectangles have 2 pairs of parallel sides.
1. All rectangles are parallelograms.
2. Parallelograms have exactly 1 pair of parallel sides.
3. So, all rectangles have exactly 1 pair of parallel sides.

Answers

The correct statement is:

Only some rectangles are parallelograms.

Parallelograms have 2 pairs of parallel sides.

The only rectangles with exactly one pair of parallel sides are some of them.

This statement is true. A quadrilateral having opposing sides that are parallel is known as a parallelogram. In the case of a rectangle, all four angles are right angles, and opposite sides are equal in length. Therefore, a rectangle can be considered a special type of parallelogram.

However, not all parallelograms are rectangles because parallelograms can have angles that are not right angles.

So, while all rectangles are parallelograms, not all parallelograms are rectangles. Thus, only some rectangles have two pairs of parallel sides.

for such more question on rectangles

https://brainly.com/question/20339811

#SPJ8

60+40[tex]60+40[/tex]

Answers

60 + 40 = 100

100 is the answer

Integrate e^(1-3x) dx with upper limit 1 and lower limit-1

Answers

After getting the integration  [tex]e^{(1-3x)} dx[/tex] with upper-limit 1 and lower-limit -1, we get [tex]\frac{-1}{3}[e^{-2}-e^{4}][/tex]

We know,

[tex]\int\limits^a_{b} {f(x)} \, dx[/tex] = [tex][F(x)]\limits^a_b[/tex]=F(a)- F(b).

Where,

a⇒Upper limit.

b⇒Lower limit,

f(x)⇒Any function of x.

F(x)⇒ [tex]\int {f(x)}[/tex] gives its antiderivative F(x).

Now here,

a is given as +1, and b is given as -1.

f(x)= [tex]e^{(1-3x)}[/tex].

Suppose, 1-3x =t.

∴ -3dx =dt.[By applying derivative rule]

Now,[tex]\int\limits e^{(1-3x)} dx[/tex]

=[tex]\int e^t.(\frac{-1}{3} ) dt[/tex]

=[tex]-\frac{1}{3} \int {e^t} dt[/tex].

=[tex]-\frac{e^t}{3}dt[/tex]

=[tex]\frac{1}{3}e^{(1-3x)}[/tex]

∴,[tex]\int\limits e^{(1-3x)} dx[/tex]  =[tex]\frac{1}{3}e^{(1-3x)}[/tex].

So,[tex]\int\limits^1_{-1} e^{(1-3x)} \, dx[/tex]

=- [tex][\frac{1}{3}e^{(1-3x)}]^1_{-1}[/tex]

=[tex]\frac{-1}{3}[e^{(1-3)}-e^{(1+3)}][/tex]

=[tex]\frac{-1}{3}[e^{-2}-e^{4}][/tex]

Learn more about Intergrals,

https://brainly.com/question/27419605

write an equation of the form y=mx for the line shown below (-1,4)

Answers

The equation of the Line of the form y = mx is y = -x + 3.

To write an equation of the form y = mx for the line shown below (-1,4), we need to determine the slope (m) of the line first.

Let (x₁, y₁) = (-1, 4) be a point on the line. Now let's find another point on the line. Let's say we have another point (x₂, y₂) = (1, 2).The slope (m) of the line can be calculated using the formula:m = (y₂ - y₁) / (x₂ - x₁)Substituting the values,

we get:m = (2 - 4) / (1 - (-1))= -2 / 2= -1

Now that we know the slope of the line, we can use the point-slope form of the equation of a line to write the equation of the line:y - y₁ = m(x - x₁)Substituting the values, we get:y - 4 = -1(x - (-1))y - 4 = -1(x + 1)y - 4 = -x - 1y = -x - 1 + 4y = -x + 3

Therefore, the equation of the line is y = -x + 3 in slope-intercept form. Since the question specifically asks for the equation of the form y = mx, we can rewrite the equation in this form by factoring out the slope:y = -x + 3y = (-1)x + 3

Thus, the equation of the line of the form y = mx is y = -x + 3.

For more questions on Line.

https://brainly.com/question/280216

#SPJ8

the drawing shows an isosceles triangle

40 degrees


can you find the size of a

Answers

Angle "a" in the given isosceles triangle is 40 degrees.

To find the size of angle "a" in the isosceles triangle with a 40-degree angle, we can use the properties of isosceles triangles. In an isosceles triangle, the two equal sides are opposite the two equal angles.

Since the given angle is 40 degrees, we know that the other two angles in the triangle are also equal. Let's call these angles "b" and "c." Therefore, we have:

b = c

Since the sum of the angles in a triangle is always 180 degrees, we can write the equation:

40 + b + c = 180

Since b = c, we can rewrite the equation as:

40 + b + b = 180

Combining like terms, we have:

2b + 40 = 180

Subtracting 40 from both sides, we get:

2b = 140

Dividing both sides by 2, we find:

b = 70

Therefore, both angles "b" and "c" are 70 degrees.

Now, we can find angle "a" by subtracting the sum of angles "b" and "c" from 180 degrees:

a = 180 - (b + c)

= 180 - (70 + 70)

= 180 - 140

= 40

For more such questions on isosceles triangle visit:

https://brainly.com/question/29793403

#SPJ8

Outside temperature over a day can be modelled as a sinusoidal function. Suppose you know the high temperature for the day is 80 degrees and the low temperature of 50 degrees occurs at 5 AM. Assuming t is the number of hours since midnight, find an equation for the temperature, D, in terms of t.

Answers

To find an equation for the temperature, D, in terms of t, we can use the properties of a sinusoidal function to model the temperature variation over the day.

Given:

High temperature: 80 degrees

Low temperature occurs at 5 AM (t = 5)

t is the number of hours since midnight

Let's assume a sinusoidal function of the form:

D = A * sin(B * t + C) + Dc

where:

A represents the amplitude (half the difference between the high and low temperatures)

B represents the frequency (how many cycles occur over a 24-hour period)

C represents the phase shift (how much the function is shifted horizontally)

Dc represents the vertical shift (the average temperature throughout the day)

We can determine the values of A, B, C, and Dc based on the given information.

Amplitude (A):

The amplitude is half the difference between the high and low temperatures:

A = (80 - 50) / 2

= 30 / 2

= 15 degrees

Frequency (B):

Since we want the temperature to complete one cycle over a 24-hour period, the frequency can be calculated as:

B = 2π / 24

Phase Shift (C):

Since the low temperature occurs at 5 AM (t = 5), the function should be shifted horizontally by 5 hours. To convert this to radians, we multiply by (2π / 24):

C = 5 * (2π / 24)

Vertical Shift (Dc):

The average temperature throughout the day is the midpoint between the high and low temperatures:

Dc = (80 + 50) / 2

= 130 / 2

= 65 degrees

Now we can put all the values together to obtain the equation for the temperature, D, in terms of t:

D = 15 * sin((2π / 24) * t + (5 * 2π / 24)) + 65

Simplifying further:

D = 15 * sin((π / 12) * t + (π / 12)) + 65

Therefore, the equation for the temperature, D, in terms of t is:

D = 15 * sin((π / 12) * t + (π / 12)) + 65.

The effectiveness of a blood-pressure drug is being investigated. An experimenter finds that, on average, the reduction in systolic blood pressure is 23.5 for a sample of size 775 and standard deviation 12.2. Estimate how much the drug will lower a typical patient's systolic blood pressure (using a 95% confidence level). Enter your answer as a tri-linear inequality accurate to one decimal place (because the sample statistics are reported accurate to one decimal place).

Answers

The 95% confidence interval for the effectiveness of the blood-pressure drug is given as follows:

[tex]22.6 < \mu < 24.4[/tex]

How to obtain the confidence interval?

The mean, the standard deviation and the sample size for this problem, which are the three parameters, are given as follows:

[tex]\overline{x} = 23.5, \sigma = 12.2, n = 775[/tex]

Looking at the z-table, the critical value for a 95% confidence interval is given as follows:

z = 1.96.

The lower bound of the interval is then given as follows:

[tex]23.5 - 1.96 \times \frac{12.2}{\sqrt{775}} = 22.6[/tex]

The upper bound of the interval is then given as follows:

[tex]23.5 + 1.96 \times \frac{12.2}{\sqrt{775}} = 24.4[/tex]

More can be learned about the z-distribution at https://brainly.com/question/25890103

#SPJ1

help me please. identify the errors in the proposed proofs

Answers

The errors in the proposed statement to prove by contradiction that 3·√2 - 7 is an irrational number, is the option;

To apply the definition of rational, a and b must be integers

What is proving by contradiction?

Proving by contradiction is an indirect method of proving a fact or a reductio ad absurdum, which is a method of proving a statement by the assumption that the opposite of the statement is true, then showing that a contradiction is obtained from the assumption.

The definition of rational numbers are numbers that can be expressed in the form a/b, where a and b are integers

The assumption that 3·√2 - 7 is a rational number indicates that we get;

3·√2 - 7 = a/b, where a and b are integers

Therefore, the error in the method used to prove that 3·√2 - 7 is an irrational number is the option; To apply the definition of rational, a and b must be integers. This is so as the value 3·√2 - 7 is a real number, which is also an irrational number, thereby contradicting the supposition.

Learn more on rational numbers here: https://brainly.com/question/31330500

#SPJ1

PLSSS HELP 13 POINTS

Answers

The equation of the line perpendicular to  y = 2 / 3 x - 4 and passes through (6, -2) is y = - 3 / 2x + 7.

How to represent equation in slope intercept form?

The equation of a line can be represented in slope intercept form as follows:

y = mx + b

where

m = slope of the lineb = y-intercept

The slopes of perpendicular lines are negative reciprocals of one another.

Therefore, the slope of the line perpendicular to y = 2 / 3 x - 4 is - 3 / 2.

Hence, let's find the line as its passes through (6, -2).

Therefore,

y = - 3 / 2 x + b

-2 = - 3 / 2(6) + b

-2  = -9 + b

b = -2 + 9

b = 7

Therefore, the equation of the line is y = - 3 / 2x + 7.

learn more on slope intercept form here: https://brainly.com/question/29146348

#SPJ1

[tex]\lim_{x,y \to \infty} \frac{x+y}{x^{2} +y^{2}-xy }[/tex]

Answers

To evaluate the limit [tex]\sf \lim_{x,y \to \infty} \frac{x+y}{x^{2} +y^{2}-xy} \\[/tex], we can analyze the behavior of the expression as both [tex]\sf x \\[/tex] and [tex]\sf y \\[/tex] approach infinity.

Let's consider the numerator [tex]\sf x + y \\[/tex] and the denominator [tex]\sf x^{2} + y^{2} - xy \\[/tex] separately.

For the numerator, as both [tex]\sf x \\[/tex] and [tex]\sf y \\[/tex] approach infinity, their sum [tex]\sf x+y \\[/tex] will also approach infinity.

For the denominator, we can rewrite it as [tex]\sf (x-y)^2 + 2xy \\[/tex]. As [tex]\sf x[/tex] and [tex]\sf y[/tex] approach infinity, the terms [tex]\sf (x-y)^2 \\[/tex] and [tex]\sf 2xy \\[/tex] will also approach infinity. Therefore, the denominator will also approach infinity.

Now, let's consider the entire fraction [tex]\sf \frac{x+y}{x^{2} +y^{2}-xy} \\[/tex]. Since both the numerator and denominator approach infinity, we have an indeterminate form of [tex]\sf \frac{\infty}{\infty} \\[/tex].

To evaluate this indeterminate form, we can apply techniques such as L'Hôpital's rule or algebraic manipulations. However, in this case, we can simplify the expression further.

By dividing both the numerator and denominator by [tex]\sf x^{2} \\[/tex], we get:

[tex]\sf \lim_{x,y \to \infty} \frac{\frac{x}{x^{2}} + \frac{y}{x^{2}}}{1 + \frac{y^{2}}{x^{2}} - \frac{xy}{x^{2}}} \\[/tex]

As [tex]\sf x[/tex] approaches infinity, the terms [tex]\sf \frac{x}{x^{2}} \\[/tex] and [tex]\sf \frac{y}{x^{2}} \\[/tex] both approach zero. Similarly, the term [tex]\sf \frac{y^{2}}{x^{2}}[/tex] and [tex]\sf \frac{xy}{x^{2}} \\[/tex] also approach zero.

Therefore, the limit simplifies to:

[tex]\sf \lim_{x,y \to \infty} \frac{0 + 0}{1 + 0 - 0} = \frac{0}{1} = 0 \\[/tex]

Hence, the limit [tex]\sf \lim_{x,y \to \infty} \frac{x+y}{x^{2} +y^{2}-xy} \\[/tex] is equal to 0.

Find the variance of the data. 198, 190, 245, 211, 193, 193
bar x=205
Variance(o2)=?
Round to the nearest tenth.

Answers

The variance of the data-set in this problem is given as follows:

σ² = 366.3.

How to obtain the variance of the data-set?

The variance of a data-set is calculated as the sum of the differences squared between each observation and the mean, divided by the number of values.

The mean for this problem is given as follows:

205.

Hence the sum of the differences squared is given as follows:

SS = (198 - 205)² + (190 - 205)² + (245 - 205)² + (211 - 205)² + (193 - 205)² + (193 - 205)²

SS = 2198.

There are six values, hence the variance is given as follows:

σ² = 2198/6

σ² = 366.3

More can be learned about the variance of a data-set at https://brainly.com/question/24298037

#SPJ1

six people want equally share 1 1/2 pizzas. how much pizza does each person get?

Answers

Each person gets 4 slices of pizza.

Given there are 6 people who want to equally share 1 1/2 pizzas, we can set up an equation by first converting 1 1/2 to an improper fraction:

1 1/2 = ((2 • 1) + 1) / 2
1 1/2 = 3/2

Now, we can divide 6 by 3/2:

6 / (3/2) = (6 • 2) / 3
12 / 3 = 4

Therefore, 6 / 1 1/2 = 4. This means each person receives 4 slices of pizza.

How do I find GBA and show all the work

Answers

Answer:

Angle ACB = 44°

There are two ways to solve it. Both are right

Solution number 1

From triangle ABC

angle BAC = 180°-(102° +44°) = 36°

Because BG is parallel with AC

Then angle GBA = angle BAC = 34°

Another solution

The sum of angles in the shape AGBC = 360°

So angle GBC = 360 - (90 + 90 + 44 + 102) = 34°

how to draw the 6th term .

Answers

To draw the 6th term, represent it visually within the context of the pattern or sequence from which it is derived.

To draw the 6th term, we need to understand the context or pattern from which the term is derived.

Drawing the term usually involves representing the elements or characteristics of the pattern in a visual form.

Without specific information about the pattern, we can provide a general approach to drawing the 6th term.

Identify the Pattern:

Determine the sequence or pattern from which the 6th term is derived.

It could be a numerical sequence, a geometric pattern, or any other pattern.

For example, if the pattern is a number sequence of multiples of 3, the first few terms would be 3, 6, 9, 12, 15, and so on.

Visualize the Pattern: Based on the identified pattern, visualize how the elements change or progress from term to term.

This could involve drawing a diagram, a graph, or any visual representation that captures the pattern.

Consider using a coordinate grid, a number line, or any other suitable visual aid.

Locate the 6th Term:

Use the information from the pattern and the visualization to determine the specific position or value of the 6th term.

In our example of multiples of 3, the 6th term would be 18.

Draw the 6th Term: Finally, represent the 6th term in your chosen visual form.

This could mean marking the position on a number line, plotting a point on a graph, or incorporating the value into a diagram.

Note that the specific method of drawing the 6th term will depend on the nature of the pattern and the context in which it is given.

Providing more details about the pattern would allow for a more accurate and specific visual representation of the 6th term.

For similar question on sequence.

https://brainly.com/question/28036578

#SPJ8

1
Select the correct answer.
The surface area of a cone is 250 square centimeters. The height of the cone is double the length of its radius.
What is the height of the cone to the nearest centimeter?
O A.
OB.
O C.
10 centimeters
15 centimeters
5 centimeters
OD. 20 centimeters
Reset
Next

Answers

Answer:

D. 20 centimeters

Step-by-step explanation:

Surface area of a cone = surface area of a circle = pi r^2

250 = pi r^2

[tex]r = \sqrt{ \frac{250}{2} } = 5 \sqrt{5} \: cm[/tex]

Because the height (h) of the cone is double the length of its radius

Then

h = 2r

[tex]h \: = 2 \times 5 \sqrt{5} = 10 \sqrt{5} = 22.36 \: cm[/tex]

So it'll equal approximate 20 cm

Other Questions
3. Which one of the following marketing strategies is not the core element for the existence of Entrepreneurs? A. Product B. Price C. Promotion D. Trade mark Which one of the following statements does not reflect the contribution of entrepreneurs to the society? A. Employment B. Independence 5. The ultimate goal of marketing in small business context is A Facilitating effective exchange B. Facilitating money generation C. Balanced development D. Equitable distribution of wealth C. Facilitating construction D. Facilitate globalization 6. The stage of new product development at which product prototype or sample may be developed is A. Product development C. Commercialization B. Market testing D. Equitable distribution of wealth 7. An entrepreneur's belief that his destiny is within his control and external forces will have little influence is demonstrated by one of the following characteristics A. Risk taking Internal locus of control B. Hard working D External locus of control 8. Which one of the following statements is incorrect about entrepreneur? A. Who has the ability to see and evaluate business opportunities B. Who has the ability to gather the necessary resources, to take advantage of opportunities C. Who has the ability to take risk and initiate appropriate action to ensure success D. None of the above 9. The marketing strategy that adds place and time utility is A. Product development C Pricing B. Distribution D. Promotion 10 The first step in developing a written business plan is A. Location selection B. Financial statement preparation 11. Which of the following statement is/are not a success factor for small business? A. Effective and efficient management B. Proper location, layout and size 12. From the core concepts of marketing, customer ability and willingness to buy can be represented A Need X B. Demand C. Identifying and evaluating a business idea D. All of the above C. Adequate finance D. Ambiguous objectives C Want D. Transaction & commence sites such as Amazon & Endless. Page 3. An equation for the graph shown to the right is: 4 y=x(x-3) C. y=x(x-3) b. y=x(x-3)) d. y=-x(x-3) 4. The graph of the function y=x is transformed to the graph of the function y=-[2(x + 3)]* + 1 by a. a vertical stretch by a factor of 2, a reflection in the x-axis, a translation of 3 units to the right, and a translation of 1 unit up b. a horizontal stretch by a factor of 2, a reflection in the x-axis, a translation of 3 units to the right, and a translation of 1 unit up c. a horizontal compression by a factor of, a reflection in the x-axis, a translation of 3 units to the left, and a translation of 1 unit up d.a horizontal compression by a factor of, a reflection in the x-axis, a translation of 3 units to the right, and a translation of 1 unit up 5. State the equation of f(x) if D = (x = Rx) and the y-intercept is (0.-). 2x+1 x-1 x+1 f(x) a. b. d. f(x) = 3x+2 2x + 1 3x + 2 - 3x-2 3x-2 6. Use your calculator to determine the value of csc 0.71, to three decimal places. b. a. 0.652 1.534 C. 0.012 d. - 80.700 At the end of the current year, Accounts Receivable has a balance of $105,680; Allowance for Doubtful Accounts has a debit balance of $4,741; and sales for the year total $927,000. Bad debt expense is estimated at 2% of sales. a. Determine the amount of the adjusting entry for bad debt expense. b. Determine the adjusted balances of Accounts Receivable, Allowance for Doubtful Accounts, and Bad Debt Expense. C. Determine the net realizable value of accounts receivable. A swimming pool with a rectangular surface 20.0 m long and 15.0 m wide is being filled at the rate of 1.0 m/min. At one end it is 1.1 m deep, and at the other end it is 3.0 m deep, with a constant slope between ends. How fast is the height of water rising when the depth of water at the deep end is 1.1 m? Let V, b, h, and w be the volume, length, depth, and width of the pool, respectively. Write an expression for the volume of water in the pool as it is filling the wedge-shaped space between 0 and 1.9 m, inclusive. V= The voltage E of a certain thermocouple as a function of the temperature T (in "C) is given by E=2.500T+0.018T. If the temperature is increasing at the rate of 2.00C/ min, how fast is the voltage increasing when T = 100C? GIZ The voltage is increasing at a rate of when T-100C. (Type an integer or decimal rounded to two decimal places as needed.) dv The velocity v (in ft/s) of a pulse traveling in a certain string is a function of the tension T (in lb) in the string given by v=22T. Find dt dT if = 0.90 lb/s when T = 64 lb. dt *** Differentiate v = 22T with respect to time t. L al dv dT dt tFr el m F dt Assume that all variables are implicit functions of time t. Find the indicated rate. dx dy x +5y +2y=52; = 9 when x = 6 and y = -2; find dt dt dy (Simplify your answer.) ... m al Assume that all variables are implicit functions of time t. Find the indicated rate. dx dy x + 5y + 2y = 52; =9 when x = 6 and y = -2; find dt dt dy y = (Simplify your answer.) ... a healthy adult male has approximately __________ liters of blood. Can University tuition fees be securitised? Explain your answerin 250 words or less, and use at least 2 academic references. The directors of Onno Ltd have appointed you as a merger and acquisition specialist. They are considering the acquisition of Otto Ltd. You are to advise them whether or not to proceed with the project. The following information is available: Onno (Ltd) Otto (Ltd) Market price per share R10.00 R8.00 Earnings per share R3.00 R2.40 No. of shares issued 2 million 0.5 million Cash payment to Otto Ltd = R12 million. Synergy benefits of R10 million will accrue through the acquisition. Otto Ltd have just had their assets re-valued and the valuation has appreciated quite significantlyRequired:Calculate the post-acquisition increase/decrease price of the share (2)Assume the acquisition is based on earnings per share:Calculate the exchange ratio based on earnings per share (3)Calculate the total number of shares in the proposed acquisition (2)Calculate the post-acquisition earnings per share (4) The position of a body over time t is described by What kind of damping applies to the solution of this equation? O The term damping is not applicable to this differential equation. O Supercritical damping O Critical damping O Subcritical damping D dt dt +40. Find the values of c, c2, and c3 so that c (5, 5,-2) + c (10,-1,0) + c3 (-5,0,0) = (-10,-1,-6). Henderson Industries Inc.'s stock is currently selling at $22.60 per share. Sharon Jacobs, the CEO, has options to buy 220,000 shares at $25.50 per share that expire at the end of this year. Sharon feels that if the traditional accounting method is used, implementing the deferred payment sales program will push the stock's price about half way toward the level it was at two years ago which was about $50.00. If Sharon Jacobs is also a founder of the company and has retained 9 million shares of its stock, how much of a difference will the auditors' decision make in her personal wealth outside of the stock option? Enter your answer in millions. For example, an answer of $1.2 million should be entered as 1.2, not 1,200,000. Round your answer to one decimal place. $ A car is moving on a straight road from Kuantan to Pekan with a speed of 115 km/h. The frontal area of the car is 2.53 m. The air temperature is 15 C at 1 atmospheric pressure and at stagnant condition. The drag coefficient of the car is 0.35. Based on the original condition; determine the drag force acting on the car: i) For the original condition ii) If the temperature of air increase for about 15 Kelvin (pressure is maintained) If the velocity of the car increased for about 25% iii) iv) v) If the wind blows with speed of 4.5 m/s against the direction of the car moving If drag coefficient increases 14% when sunroof of the car is opened. Determine also the additional power consumption of the car. The Following Business Transactions In General Journai Form. Identify Each Transaction By Number. You May Omit Explanations Of The Transaction. (Credit Account Titles Are Automatically Indented When The Amount Is Entered. Do Not Indent Manually. If No Entry Is Required, Select "No Entry" For The Account Titles And Find a bijective mapping from N to Z, and explain why it is bijective. Consider the state of the world right now economically, socially, technologically, environmentally, etc. Think the PESTEL analysis. In 150-200 words, what do you think is on the mind of its CEO right now? Particularly, if you selected a multi-national company, how might current events impact the different types of strategic initiatives the CEO might be considering? Given F(x, y) = (sin(x-y), -sin(x-y)) M a. Is F(x, y) conservative? b. Find the potential function f(x, y) if it exists. If a business practices tax shifting, how does this affect consumers?a. Consumers enjoy paying lower prices because the business decides to be accountable for the tax burden.b. Consumers pay higher prices because the business is passing along higher taxes to the consumer.c. Consumers tend to purchase more from this business because the business is trying to benefit the consumer.d. Consumers are not affected by tax shifting because this is the practice of paying federal income tax over an extended period of time. What is cyber wellness? marketers with luxury brands use brand extension cautiously to avoid Economics3. Using the AA-DD model, explain:(a) why a temporary increase in the money supply raises output and the exchange rate;(b) why the effects of a permanent increase in the money supply are differentfrom (a) Using the returns shown, calculate the arithmetic average returns, the variances, and the standard deviations for X and Y. Year 1: x=15%, y=20%; Year 2: X=18%, y=30%; Year 3: X= -9, y= - 16; Year 4: X=10% and Y=15%. I have answers and EXCEL chart done if you need.My average returns for X are 8.6% and for Y 13.8%.My Standard deviations are correct for X at 10.50 and for Y at 17.53.My variances are INCORRECT. X = 110.300000 an for Y 307.20000 (5 places required)Thanks. I am desperate. I do not know what I am doing wrong.I can send Excel document.Thanks.Carol